Problem with Thermodynamics. Period dependence on temparature.

In summary,homework statement states that we need to find k in this formula. F= -k x ( it's the force, which trying to return piston back to his place) And place it in this formula T = 2 Pi sqrt(m1/k)
  • #1
AudriusR
11
0

Homework Statement


http://img267.imageshack.us/img267/5830/80849533.jpg
Picture. The volume of that cylinder is V = 1l. In that cylinder we have 1g of hydrogen m=0,001 kg. The cylinder is separated in two equal parts with m1=0,005 kg and d=0.006m length piston. Imagine that we pushed that cylinder and piston in there began to swing. We need to calculate swings period dependence on temperature of the gases.

Homework Equations



We need to find k in this formula. F= -k x ( it's the force, which trying to return piston back to his place) And place it in this formula T = 2 Pi sqrt(m1/k)


The Attempt at a Solution


In my opinion that F is the difference of pressures in that cylinder.
I would calculate both pressures with this formula:
p V = m/M R T

and then I would deprive.

F would be : F = (delta)p S

But when I calculate, my V and m depends on the x, and he disappears.

In my head there are no more thoughts, so I'm asking you for help.

P.S I tried to put symbols of Pi and delta, but instead of them in the post there were question-marks. Why?
 
Last edited by a moderator:
Physics news on Phys.org
  • #2
Below I keep temperature constant, for small displacements I think that is OK. Hope the following helps.

For more accuracy you may want to expand V^2 = [V_o + dV]^2 about V_o ?
 

Attachments

  • aaaaa112.jpg
    aaaaa112.jpg
    23.8 KB · Views: 394
Last edited:
  • #3
Sorry, but I didn't get it, how to get f(x) function, when we have f(dx) ?
Or maybe it's possible to calculate period with this function?
 
  • #4
Just let dx = x.
 
  • #5
I'm really sorry for my questions, but I didn't get, how to get exact answer for Periods depence on temperature ( T(T) ). Maybe, if I don't ask you too much, you could give me another hint.
 
  • #6
AudriusR said:
I'm really sorry for my questions, but I didn't get, how to get exact answer for Periods depence on temperature ( T(T) ). Maybe, if I don't ask you too much, you could give me another hint.

On my sketch I wrote

F = [-nRTA^2dx]/v^2

let dx = x

F = [-nRTA^2x]/v^2

This is of the form,

F = kx

which you should know from the harmonic oscillator. Now F is a function of several numbers, one of them is T (all but x are roughly constant for small displacements x). The corrections go as x^2?The problem gets harder if we need to be more accurate. I don't know how far you need to go.

Good luck!
 
Last edited:
  • #7
AudriusR said:
We need to find k in this formula. F= -k x ( it's the force, which trying to return piston back to his place) And place it in this formula T = 2 Pi sqrt(m1/k)


The Attempt at a Solution


In my opinion that F is the difference of pressures in that cylinder.
I would calculate both pressures with this formula:
p V = m/M R T

and then I would deprive.

F would be : F = (delta)p S

But when I calculate, my V and m depends on the x, and he disappears.

In my head there are no more thoughts, so I'm asking you for help.

P.S I tried to put symbols of Pi and delta, but instead of them in the post there were question-marks. Why?
Assume the cylinder is insulated and that the compressions and expansions are adiabatic. You have to use the adiabatic condition PV^γ = K in order to solve this. You cannot just assume that the restoring force is linear.

AM
 
  • #8
Andrew Mason said:
Assume the cylinder is insulated and that the compressions and expansions are adiabatic. You have to use the adiabatic condition PV^γ = K in order to solve this. You cannot just assume that the restoring force is linear.

AM

I'm confused then. If my sketch of the pressure verses position of the cylinder above is close then for small displacements the sum of the two pressures will be some linear function of x (for small x)? For small displacements F = kx.

I not too sure about my value for k though.
 
  • #9
Spinnor said:
I'm confused then. If my sketch of the pressure verses position of the cylinder above is close then for small displacements the sum of the two pressures will be some linear function of x (for small x)? For small displacements F = kx.

I not too sure about my value for k though.
I am saying that the restoring force is not linear. It is proportional to -x^γ

Since the initial volumes are the same (say, V) and the pressures and temperatures are initially the same in the equilibrium position, and since the total volume is always the same:

P1(V-dV)^γ = P2(V+dV)^γ = K

Since the net pressure on the piston is the difference between the two pressures:

P1-P2 = F/A = K/(V-dV)^γ - K/(V+dV)^γ = K/(V-Ax)^γ - K/(V+Ax)^γ

Letting A be the cross-sectional area of the cylinder and L the length of the space on each side at equilibrium the net force is:

[itex]F = K(A/(AL-Ax)^γ - A/(AL+Ax)^γ) = KA^{(1-γ)}(1/(L-x) - 1/(L+x))^γ[/itex]

For x << L, (1/(L-x) - 1/(L+x))^γ ≈ -(2x/L^2)^γ

So it seems to me that:

[tex]F = - KA^{(1-\gamma)}\left(\frac{2}{L^{2}}\right)^\gamma x^\gamma = -kx^\gamma [/tex]

where [itex]k = KA^{(1-\gamma)}\left(\frac{2}{L^{2}}\right)^\gamma[/itex]

AM
 
  • #10
Andrew Mason said:
I am saying that the restoring force is not linear. It is proportional to -x^γ

Since the initial volumes are the same (say, V) and the pressures and temperatures are initially the same in the equilibrium position, and since the total volume is always the same:

P1(V-dV)^γ = P2(V+dV)^γ = K

Since the net pressure on the piston is the difference between the two pressures:

P1-P2 = F/A = K/(V-dV)^γ - K/(V+dV)^γ = K/(V-Ax)^γ - K/(V+Ax)^γ

Letting A be the cross-sectional area of the cylinder and L the length of the space on each side at equilibrium the net force is:

[itex]F = K(A/(AL-Ax)^γ - A/(AL+Ax)^γ) = KA^{(1-γ)}(1/(L-x) - 1/(L+x))^γ[/itex]

For x << L, (1/(L-x) - 1/(L+x))^γ ≈ -(2x/L^2)^γ

So it seems to me that:

[tex]F = - KA^{(1-\gamma)}\left(\frac{2}{L^{2}}\right)^\gamma x^\gamma = -kx^\gamma [/tex]

where [itex]k = KA^{(1-\gamma)}\left(\frac{2}{L^{2}}\right)^\gamma[/itex]

AM

Thank you for a detailed answer, but since given data include V and length of piston d I'm not so sure, if I can get exact result in those data. Maybe, you could suggest something?
 
  • #11
If you graph pressure on one side verses piston displacement and then expand about small displacements no matter what the function for pressure we get,

P(dx) = c1 + c2dx + c3dx^2 + ...

For small displacements pressure difference will go as dx? If we add the pressure on both sides the constants c1 will cancel and to lowest order in dx we will have for the force on the piston,

F = 2Ac2dx + higher order terms.

The constant c2 will be found via Andrews help,

"Assume the cylinder is insulated and that the compressions and expansions are adiabatic. You have to use the adiabatic condition PV^γ = K in order to solve this. ... ."
 
  • #12
Spinnor said:
If you graph pressure on one side verses piston displacement and then expand about small displacements no matter what the function for pressure we get,

P(dx) = c1 + c2dx + c3dx^2 + ...

For small displacements pressure difference will go as dx? If we add the pressure on both sides the constants c1 will cancel and to lowest order in dx we will have for the force on the piston,

F = 2Ac2dx + higher order terms.

The constant c2 will be found via Andrews help,

"Assume the cylinder is insulated and that the compressions and expansions are adiabatic. You have to use the adiabatic condition PV^γ = K in order to solve this. ... ."



So making simple mathematics, what result I should get ?

Maybe, you could look at my calculations:

F= 2Ac2dx

c2 = KA^(1-?) (2/L^2)^?

F= 2K A^(2-?) (2/L^2)^? dx

So period formula looks like this:

T= 2 ? sqrt( m1/k)

k= 2K^(2-?) (2/L^2)^?

T = 2 ? sqrt ( A^? m1 (L^?)^2/ 2^(?+1) A^2 K )

K = p1 (V + dV)^? = p2 ( V-dV)^?

Consideringt that:

p1(V+dV) + p2 (V-dV) = m/M R T ( Am I right? )

K = 2 (mRT/M)^?

And the result is

T(T) = 2 ? sqrt( V^? m1 L^? M^? / 2^(?+2) A^2 (mRT)^? )

Also there is a problem, I couldn't get result by given data ( L and A hadn't been given)

Any ideas, how to remove those and is this is a good result ?
 
  • #13
From my earlier post:

[itex]F = -kx^\gamma [/itex]

where [itex]k = KA^{(1-\gamma)}\left(\frac{2}{L^{2}}\right)^\gamma[/itex] and [itex]K = PV^\gamma[/itex] where P and V are the pressure and volume at any time eg. P0 and V0, the initial pressure and volume.

To find the relationship to temperature you would have to express the adiabatic condition in terms of T0, the initial temperature and V0, the initial volume. You do this by substituting for P0: P = nRT/V

[itex]K = P_0V_0^\gamma = nRT_0V_0^{\gamma -1}[/itex]

So the force equation becomes:

[itex]F = -nRT_0V_0^{(\gamma -1)}A^{(1-\gamma)}\left(\frac{2}{L^{2}}\right)^\gamma x^\gamma[/itex]

Since [itex]A^{(1-\gamma)} = A^{-(\gamma-1)}[/itex] we can reduce this to:

[itex]F = -nRT_0\left(\frac{V_0}{A}\right)^{(\gamma -1)}\left(\frac{2}{L^{2}}\right)^\gamma x^\gamma[/itex]

But V0/A is just the length of the chamber on each side of the piston, ie. L.

So this reduces nicely to:

[itex]F = -2^\gamma nRT_0L^{(\gamma -1)}L^{-2\gamma} x^\gamma = -2^\gamma nRT_0L^{-(\gamma + 1)}x^\gamma[/itex]

AM
 
Last edited:
  • #14
I'm very grateful for your answer
 

1. What is the problem with thermodynamics and period dependence on temperature?

The problem with thermodynamics and period dependence on temperature is that it goes against the principle of thermal equilibrium, which states that the temperature of a system will eventually reach a steady state. However, in certain cases, the period or time it takes for a system to reach its steady state can vary with temperature, causing complications in thermodynamic calculations.

2. Why is period dependence on temperature a concern in thermodynamics?

Period dependence on temperature is a concern in thermodynamics because it can lead to inaccuracies in calculations and predictions. This can be problematic in areas such as engineering and chemistry, where precise measurements and predictions are crucial.

3. What factors can cause period dependence on temperature in thermodynamic systems?

Period dependence on temperature can be caused by a variety of factors, including the presence of external forces or fluctuations, the specific properties of the materials involved, and the complexity of the system. Additionally, quantum effects can also play a role in period dependence on temperature.

4. Can period dependence on temperature be avoided in thermodynamic systems?

In some cases, period dependence on temperature can be avoided or minimized by carefully controlling external factors and using more advanced thermodynamic models. However, in certain systems, it may be an inherent property that cannot be completely eliminated.

5. How can scientists and engineers address the problem of period dependence on temperature in thermodynamics?

To address the problem of period dependence on temperature, scientists and engineers can use more sophisticated thermodynamic models and techniques, conduct further research to better understand the underlying mechanisms, and work towards developing more accurate and precise methods for calculating and predicting thermodynamic behavior.

Similar threads

  • Introductory Physics Homework Help
Replies
4
Views
1K
  • Introductory Physics Homework Help
Replies
8
Views
929
  • Introductory Physics Homework Help
Replies
3
Views
929
  • Introductory Physics Homework Help
Replies
1
Views
921
  • Introductory Physics Homework Help
Replies
5
Views
2K
  • Introductory Physics Homework Help
Replies
3
Views
958
Replies
5
Views
564
  • Introductory Physics Homework Help
Replies
5
Views
1K
  • Introductory Physics Homework Help
Replies
3
Views
961
  • Introductory Physics Homework Help
Replies
14
Views
1K
Back
Top